- PowerScore Staff
- Posts: 5972
- Joined: Mar 25, 2011
- Fri Jan 21, 2011 12:00 am
#22831
Complete Question Explanation
WeakenX. The correct answer choice is (D)
This is an excellent question, and a perfect example of how "older" LSAT questions are still entirely relevant to today's LSAT.
In this stimulus, the author tells us that although city taxes subsidize Greenville public bussing, the buses benefit outside commuters as well, so some city councilors suggest increasing fares to cover the cost of the service. However, some city councillors argue for raising the fares, on the basis that "taxes should be used primarily to benefit the people who pay them."
The question which follows the stimulus is a Weaken Except question, which means that the four incorrect answer choices will weaken the councillors' argument, and the correct answer choice will not weaken it. The first and most obvious line of attack would be to find and eliminate answers that show that the outside commuters do indeed pay taxes (and indeed several answers play on this approach) or that such a plan would have general negatives that would hurt the city on the whole. In other words, eliminate any answer that hurts the argument of the city councillors, and look for an answer that is irrelevant or strengthens the argument.
Answer choice (A): Because businesses in the city are also taxpayers, this answer shows that that raising the fares would hurt a group of taxpayers. In addition, there would be a general loss of benefit to the citizens of the city if businesses left. Thus, this answer choice weakens the argument and is incorrect.
Answer choice (B): This answer choice also weakens the argument that fares should be increased, by pointing out another detrimental result associated with the councillors' suggested plan. Thus, this answer choice weakens the argument and is incorrect.
Answer choice (C): This answer choice at first appears irrelevant because the low income residents don't pay taxes, and thus one might initially think that this answer is correct. However, the second half of the answer shows that "all city councilors agree" that these residents should be able to use the buses without disadvantage. Thus, if adopted, this answer would place the "some city councillors" from the stimulus in a contradictory position (as far as their stated goals here for low income residents to take advantage of the services vs the results of their plan which disadvantages low income residents), and that would weaken the argument. This is a very tricky and very well-placed answer (just before the correct answer), evocative of recent Logical Reasoning questions.
Answer choice (D): This is the correct answer choice. This choice fails to weaken the argument advanced in the stimulus, because voter preference about increasing local taxes is irrelevant—there is no tax increase even being suggested! The suggestion is to raise bus fares. Additionally, the opinions ("preferences") in this answer do not attack the facts cited above. Since this answer choice has no effect on the strength of the councillors' argument, this is the correct answer choice to this Weaken Except question.
Answer choice (E): This answer choice weakens the argument because it shows that those "outsiders" who commute into Greenville do in fact pay taxes to cover the services they use, and thus they would qualify as as people who should benefit from them. Note how carefully the stimulus worded the issues: they never used "residents," they simply used "people who pay them (taxes)." This answer then shows that the commuters are actually taxpayers, and consequently the city taxes would be used to benefit those who pay them. As this is the very basis of the councillors' objection to the subsidy, this answer would undermine their objection.
WeakenX. The correct answer choice is (D)
This is an excellent question, and a perfect example of how "older" LSAT questions are still entirely relevant to today's LSAT.
In this stimulus, the author tells us that although city taxes subsidize Greenville public bussing, the buses benefit outside commuters as well, so some city councilors suggest increasing fares to cover the cost of the service. However, some city councillors argue for raising the fares, on the basis that "taxes should be used primarily to benefit the people who pay them."
The question which follows the stimulus is a Weaken Except question, which means that the four incorrect answer choices will weaken the councillors' argument, and the correct answer choice will not weaken it. The first and most obvious line of attack would be to find and eliminate answers that show that the outside commuters do indeed pay taxes (and indeed several answers play on this approach) or that such a plan would have general negatives that would hurt the city on the whole. In other words, eliminate any answer that hurts the argument of the city councillors, and look for an answer that is irrelevant or strengthens the argument.
Answer choice (A): Because businesses in the city are also taxpayers, this answer shows that that raising the fares would hurt a group of taxpayers. In addition, there would be a general loss of benefit to the citizens of the city if businesses left. Thus, this answer choice weakens the argument and is incorrect.
Answer choice (B): This answer choice also weakens the argument that fares should be increased, by pointing out another detrimental result associated with the councillors' suggested plan. Thus, this answer choice weakens the argument and is incorrect.
Answer choice (C): This answer choice at first appears irrelevant because the low income residents don't pay taxes, and thus one might initially think that this answer is correct. However, the second half of the answer shows that "all city councilors agree" that these residents should be able to use the buses without disadvantage. Thus, if adopted, this answer would place the "some city councillors" from the stimulus in a contradictory position (as far as their stated goals here for low income residents to take advantage of the services vs the results of their plan which disadvantages low income residents), and that would weaken the argument. This is a very tricky and very well-placed answer (just before the correct answer), evocative of recent Logical Reasoning questions.
Answer choice (D): This is the correct answer choice. This choice fails to weaken the argument advanced in the stimulus, because voter preference about increasing local taxes is irrelevant—there is no tax increase even being suggested! The suggestion is to raise bus fares. Additionally, the opinions ("preferences") in this answer do not attack the facts cited above. Since this answer choice has no effect on the strength of the councillors' argument, this is the correct answer choice to this Weaken Except question.
Answer choice (E): This answer choice weakens the argument because it shows that those "outsiders" who commute into Greenville do in fact pay taxes to cover the services they use, and thus they would qualify as as people who should benefit from them. Note how carefully the stimulus worded the issues: they never used "residents," they simply used "people who pay them (taxes)." This answer then shows that the commuters are actually taxpayers, and consequently the city taxes would be used to benefit those who pay them. As this is the very basis of the councillors' objection to the subsidy, this answer would undermine their objection.
Dave Killoran
PowerScore Test Preparation
Follow me on X/Twitter at http://twitter.com/DaveKilloran
My LSAT Articles: http://blog.powerscore.com/lsat/author/dave-killoran
PowerScore Podcast: http://www.powerscore.com/lsat/podcast/
PowerScore Test Preparation
Follow me on X/Twitter at http://twitter.com/DaveKilloran
My LSAT Articles: http://blog.powerscore.com/lsat/author/dave-killoran
PowerScore Podcast: http://www.powerscore.com/lsat/podcast/